0
$\begingroup$

Let $(a_n)$ be the sequence of minimum values of the expression (depending on $\ell$):

\begin{equation*} a_n=\min \bigg\lbrace ((\ell+1)j-n)!\,(n-\ell j)!\quad \text{for}\; \\,\bigg\lceil\frac{n}{\ell+1}\bigg\rceil\le j\le \bigg\lfloor\frac{n}{\ell}\bigg\rfloor \bigg\rbrace % \min \big\lbrace (2j-n)!\,(n- j)!,\; \mathrm{ceil}\,\frac{n}{2}\le j\le n \big\rbrace, .\end{equation*}

My question is twofold:

Is there a closed formula for $(a_n)_{n\ge \ell}$ (for a fixed $\ell$), or a way to characterize its entries? Why?

$\endgroup$
5
  • 9
    $\begingroup$ You asked this at math.stackexchange.com/questions/180374/… some 20 hours ago. It is usually best to ask on one site and wait a bit (a few days) before reasking on the other site. When you do ask again, add links between the two instances of the question. $\endgroup$ Aug 9, 2012 at 16:42
  • $\begingroup$ You can write a program, with Maple, Magma or Matlab, and obtaining some directions for examining. Also, if you give these results here, I think it is helpful and the problem will be clearer. $\endgroup$
    – Shahrooz
    Aug 9, 2012 at 17:35
  • $\begingroup$ This is for fixed l, (in my view) a fairly simple optimization, and is equivalent to the minimum of a set of two values. I would like more motivation, an idea of what you have tried, and either admission that it is homework or a real good reason why I should not let you solve it yourself. Gerhard "Willing To Give Another Hint" Paseman, 2012.08.09 $\endgroup$ Aug 9, 2012 at 18:16
  • $\begingroup$ I posted a similar question at StackExchange: math.stackexchange.com/questions/180374/… $\endgroup$ Aug 10, 2012 at 17:23
  • $\begingroup$ One thing that bothers me about the question is that it is ill defined. For example, a_(l+2) does not exist for most values of l, and this is true of some other indices. Further, a computer search using valid pairs of (l,n) should make it clear which values of j will produce the desired minimum, and such j can be easily described. Because of the nature of the description, I suspect no nice form exists for the general term. Gerhard "Ask Me About System Design" Paseman, 2012.08.10 $\endgroup$ Aug 10, 2012 at 18:15

0

Your Answer

By clicking “Post Your Answer”, you agree to our terms of service and acknowledge you have read our privacy policy.

Browse other questions tagged or ask your own question.